Functional Analysis, Show that the range of a bounded linear operator

Click For Summary
The discussion focuses on demonstrating that the range of a bounded linear operator T from l^∞ to l^∞ is not necessarily closed. A specific operator is defined, where the sequence is transformed by dividing by its index. The challenge is to identify an element y in l^∞ that is not in the range of T, with y = (1, 1, ...) serving as a candidate. The difficulty lies in constructing a convergent sequence from the range that approaches y, as bounded sequences must eventually decrease to zero. The conversation highlights the need for a different approach to find a limit point of R(T) that does not belong to R(T).
Eduardo
Messages
2
Reaction score
0

Homework Statement


Show that the range \mathcal{R}(T) of a bounded linear operator T: X \rightarrow Y is not necessarily closed.
Hint: Use the linear bounded operator T: l^{\infty} \rightarrow l^{\infty} defined by (\eta_{j}) = T x, \eta_{j} = \xi_{j}/j, x = (\xi_{j}).

Homework Equations





The Attempt at a Solution


My idea was to find an element y \in l^{\infty} that does not belong to the range and then try to build a convergent sequence in \mathcal{R}(T) that has limit y. The element y = (1, 1, \ldots) satisfy the criteria because T^{-1}y = \{ x\}, with x = (\xi_{j}), \xi_{j} = j, but, clearly, x \not\in l^{\infty}, therefore, y \not\in \mathcal{R}(T). The problem arise when I try to build the sequence, because (T x_{m}) with x_{m} \in l^{\infty} cannot converge to y. Briefly, my problem is that I can´t find a limit point of \mathcal{R}(T) that doesn´t belong to \mathcal{R}(T).
 
Physics news on Phys.org
Think from the other direction. Since x needs to be bounded, the elements in Tx must decrease to zero eventually. Which means that any element that's on the boundary of the image set must also have its elements converge to zero. Can you find a sequence nk that converges to zero, such that knk is unbounded?
 
Just, logged into thank you, Office Shredder, for your quick and helpful answer. Wich was very nice because it was precisely what I needed, to unblock my fixed ideas o a change of approach to face the problem and let me think or find an answer instead of just give me the result inmediately. Thanks again.
 
who did you find the find a limit point of
R(T)
that doesn´t belong to
R(T)?
 
Question: A clock's minute hand has length 4 and its hour hand has length 3. What is the distance between the tips at the moment when it is increasing most rapidly?(Putnam Exam Question) Answer: Making assumption that both the hands moves at constant angular velocities, the answer is ## \sqrt{7} .## But don't you think this assumption is somewhat doubtful and wrong?

Similar threads

  • · Replies 13 ·
Replies
13
Views
2K
  • · Replies 2 ·
Replies
2
Views
1K
  • · Replies 43 ·
2
Replies
43
Views
4K
  • · Replies 14 ·
Replies
14
Views
2K
  • · Replies 8 ·
Replies
8
Views
1K
  • · Replies 1 ·
Replies
1
Views
1K
Replies
8
Views
2K
Replies
9
Views
2K
  • · Replies 2 ·
Replies
2
Views
2K
  • · Replies 1 ·
Replies
1
Views
2K